2014-2015/1S/DM/DM_0528/13_DM_0528.tex

177 lines
5.8 KiB
TeX

\documentclass[a4paper,12pt, table]{/media/documents/Cours/Prof/Enseignements/2014-2015/Archive/2014-2015/tools/style/classDS}
\usepackage{/media/documents/Cours/Prof/Enseignements/2014-2015/Archive/2014-2015/2014_2015}
% Title Page
\titre{DM7}
% \seconde \premiereS \PSTMG \TSTMG
\classe{\premiereS}
\date{28 mai 2015}
%\duree{1 heure}
\sujet{13}
% DS DSCorr DM DMCorr Corr
\typedoc{DM}
%\printanswers
\begin{document}
\maketitle
Le barème est donné à titre indicatif, il pourra être modifié. Vous rendrez le sujet avec la copie.
\begin{questions}
\question
\begin{parts}
\part Dessiner un cercle trigonométrique et y placer les angles suivants (détailler les calculs si vous utilisez la mesure principale de l'angle)
\begin{multicols}{2}
\begin{parts}
\part $\alpha = \frac{33\pi}{6}$
\part $\beta= \frac{34\pi}{4}$
\part $\delta = \frac{17\pi}{6}$
\part $\sigma = \frac{26\pi}{6}$
\end{parts}
\end{multicols}
\begin{solution}
\begin{tikzpicture}
\cercleTrigo
\draw (990.0:1) node[rotate = 990.0] {-} node[above] {$\alpha$};
\draw (1530.0:1) node[rotate = 1530.0] {-} node[below] {$\beta$};
\draw (510.0:1) node[rotate = 510.0] {-} node[right] {$\delta$};
\draw (780.0:1) node[rotate = 780.0] {-} node[left] {$\sigma$};
\end{tikzpicture}
\end{solution}
\part On pose $||\vec{u}|| = 2 $, $||\vec{v}|| = 1 $ et $\vec{u}.\vec{v} = -1.0$ calculer les quantités suivantes
\begin{multicols}{2}
\begin{subparts}
\subpart $(\vec{u} - 5 \vec{v})(\vec{v} + 5 \vec{u})$
\subpart $||5\vec{u} - 5 \vec{v}||$
\end{subparts}
\end{multicols}
\end{parts}
\question
\begin{parts}
\part Déterminer le domaine de définition des fonctions suivantes
% Il y aura toujours 2 racines
% Il y aura toujours 2 racines
% Il y aura toujours une valeur interdite à ajouter
\begin{multicols}{3}
\begin{subparts}
\subpart $f:x \mapsto \dfrac{1}{4 x^{ 2 } - 8 x - 5}$
\subpart $g:x\mapsto \dfrac{1}{\sqrt{x} - 9}$
\subpart $h:x \mapsto \sqrt{10 x^{ 2 } - 5 x - 4}$
\end{subparts}
\end{multicols}
\begin{solution}
\begin{enumerate}
\item
On constate que $f$ est une fonction de la forme
\begin{eqnarray*}
f(x) = \frac{1}{u(x)} &\mbox{ avec }& u(x) = 4 x^{ 2 } - 8 x - 5
\end{eqnarray*}
Comme $u(x)$ est un polynôme, son domaine de définition est $D_u = \R$. Il faut maintenant déterminer les valeurs de $x$ tels que $u(x) = 0$.
On résout l'équation $4 x^{ 2 } - 8 x - 5 = 0$:
On commence par calculer le discriminant de $P(x) = 4 x^{ 2 } - 8 x - 5$.
\begin{eqnarray*}
\Delta & = & b^2-4ac \\
\Delta & = & ( -8 )^{ 2 } - 4 \times 4 \times ( -5 ) \\
\Delta & = & 64 - 4 \times ( -20 ) \\
\Delta & = & 64 - ( -80 ) \\
\Delta & = & 144
\end{eqnarray*}
comme $\Delta = 144 > 0$ donc $P$ a deux racines
\begin{eqnarray*}
x_1 & = & \frac{-b - \sqrt{\Delta}}{2a} = \frac{-8 - \sqrt{144}}{2 \times 4} = - \frac{1}{2} \\
x_2 & = & \frac{-b + \sqrt{\Delta}}{2a} = \frac{-8 + \sqrt{144}}{2 \times 4} = \frac{5}{2}
\end{eqnarray*}
Les solutions de l'équation $4 x^{ 2 } - 8 x - 5 = 0$ sont donc $\mathcal{S} = \left\{ - \frac{1}{2}; \frac{5}{2} \right\}$
Donc finalement, $f$ est définie sur $D_f = \R \backslash \left\{ - \frac{1}{2}, \frac{5}{2} \right\}$.
\end{enumerate}
\end{solution}
\part Soit $f$ un fonction définie par
\begin{eqnarray*}
f:x\mapsto \frac{- 5 x^{ 2 } - 2 x + 2}{8 x + 10}
\end{eqnarray*}
\begin{subparts}
\subpart Déterminer le domaine de définition de $f$
\subpart Démontrer que la dérivé de $f$ est $f('x) = \dfrac{- 40 x^{ 2 } - 100 x - 36}{(8 x + 10)^2}$
\subpart Étudier le signe de $f$.
\subpart Calculer l'équation de la tangente à $\mathcal{C}_f$ (la courbe représentative de $f$) au point d'abscisse $x = 1$.
\end{subparts}
\end{parts}
\question
Soit $(u_n)$ la suite définie sur $\N$ par
\begin{eqnarray*}
u_0 = 1 \hspace{2cm} u_{n+1} = 9 u_n + 8
\end{eqnarray*}
\begin{parts}
\part Calculer les 4 premiers termes de la suite $(u_n)$.
\part On pose $v_n = u_n + 1$.
\begin{subparts}
\subpart Calculer les 4 premiers termes de la suite $(v_n)$
\subpart Démontrer que $(v_n)$ est géométrique de raison $q = 9$.
\subpart En déduire l'expression explicite de $(v_n)$.
\subpart En déduire que l'expression explicite de $(u_n)$ est $u_n = 2 \times 9^{ n } - 1$
\end{subparts}
\part Écrire un algorithme qui prend en argument un rang \texttt{n} et qui renvoie la la valeur de $v_n$ (vous ne pouvez pas utiliser la formule explicite)
\part Écrire un algorithme qui prend en argument un rang \texttt{n} et qui renvoie la la valeur de $u_0 + u_1 +\cdots + u_n$ ( vous pouvez utiliser la formule explicite)
\end{parts}
\end{questions}
\end{document}
%%% Local Variables:
%%% mode: latex
%%% TeX-master: "master"
%%% End: